0 Daumen
1,8k Aufrufe

ich habe eine Frage zu einer Aufgabe.

Finden Sie für jede der folgenden prädikatenlogischen Formeln und der angegebenen
umgangssprachlichen Bedeutung jeweils eine Interpretation, welche die Formel wahr
werden lässt. Verwenden Sie dabei die angegebenen Grundbereiche ω.


1. ∀x∃y∃z(S(x, b) → Q(x, y, z) ∧ ∀v(R(v, y) → T (v, y))), ω = N
„Jede natürliche Zahl n ≥ 2 hat mindestens einen Primfaktor.“


Mein Tutor sagte, dass ich lediglich die erste Voraussetzung jeder Implikation falsch setzen muss, um die Interpretation wahr werden zu lassen.

Kann ich nun einfach z.B. folgende Interpretation machen?

1.

I(S):= {(x,b) ∈ ℕ² | x<2 und b=2 und x>b}

Damit wird die Implikation halt immer korrekt und alles danach irrelevant.

Kann ich dann einfach irgendwas in die anderen Terme einsetzen ohne groß nachzudenken?

I(Q):= {(x,y,z)∈ ℕ³|x=1 und y=2 und z=3}


Kommt mir irgendwie nicht nach einer guten Vorgehensweise vor, aber so wurde es mir erklärt, falls ich es nicht falsch verstanden habe.

Da frage ich lieber nochmal lieber nach.

Avatar von

1 Antwort

+1 Daumen
 
Beste Antwort
I(S):= {(x,b) ∈ ℕ² | x<2 und b=2 und x>b}

Dann ist I(S) = ∅.

Finden Sie für jede der folgenden prädikatenlogischen Formeln und der angegebenen
umgangssprachlichen Bedeutung jeweils eine Interpretation, welche die Formel wahr
werden lässt.

Gemeint ist wohl, dass du S, Q, R, T und b so interpretieren sollst, dass die Aussage „Jede natürliche Zahl n ≥ 2 hat mindestens einen Primfaktor.“ entsteht.

Tipp: Wenn du I(S) = {(n, m) ∈ ω×ω | n ≥ m} und I(b) = 2 wählst, dann sagt

        S(x, b)

schon mal aus, dass

        x ≥ 2

ist.

Avatar von 105 k 🚀

Danke.

Sind folgende Interpretationen korrekt?

Ich habe die Vermutung, dass ich bei I(R) noch hinzufügen muss, dass es die einzigen Teiler sind, aber ich bin mir gerade nicht sicher wie. Oder kann ich das einfach in Worten dahin schreiben?

9C4877E5-157E-471D-B048-4610E2E945BD.jpeg

I(R) := {(x, y) ∈ ω2 | x|y ∧ x ≠ 1 }

Das y|y ist immer erfüllt.

I(T) := {(x, y) ∈ ω2 | x = y }

Danke bro. Jetzt habe ich es verstanden.

Ein anderes Problem?

Stell deine Frage

Willkommen bei der Mathelounge! Stell deine Frage einfach und kostenlos

x
Made by a lovely community